Q19

User avatar
 
ManhattanPrepLSAT1
Thanks Received: 1909
Atticus Finch
Atticus Finch
 
Posts: 2851
Joined: October 07th, 2009
 
 
 

Q19

by ManhattanPrepLSAT1 Mon Nov 29, 2010 6:50 pm

mturner Wrote:I am really stuck just trying to figure out the set up on this one. Based on the setup, does that mean that the same swimmer on lap 1 is also on lap 6?

On question 19, I'm stuck between A and C. Neither one appear to violate any rules.

Other than figuring out that Ortiz can't swim lap 1, I can't figure out the rest of the swimmers for the first five laps.

I'm getting frustrated here. If I could get the diagram straight, I will be in good shape. Help!
User avatar
 
ManhattanPrepLSAT1
Thanks Received: 1909
Atticus Finch
Atticus Finch
 
Posts: 2851
Joined: October 07th, 2009
 
 
 

Re: PT 37, S3, G4 - Swim Team Game

by ManhattanPrepLSAT1 Mon Nov 29, 2010 6:50 pm

tianfeng102 Wrote:
mturner Wrote:I am really stuck just trying to figure out the set up on this one. Based on the setup, does that mean that the same swimmer on lap 1 is also on lap 6?

On question 19, I'm stuck between A and C. Neither one appear to violate any rules.

Other than figuring out that Ortiz can't swim lap 1, I can't figure out the rest of the swimmers for the first five laps.

I'm getting frustrated here. If I could get the diagram straight, I will be in good shape. Help!


Based on the setup, does that mean that the same swimmer on lap 1 is also on lap 6? Yes.

On question 19, I'm stuck between A and C. Neither one appear to violate any rules. Well, C violates the last rule which states that J is immediately after O.
 
jeffshin720
Thanks Received: 0
Vinny Gambini
Vinny Gambini
 
Posts: 2
Joined: March 15th, 2012
 
 
 

Re: Q19

by jeffshin720 Thu Apr 26, 2012 12:16 am

I have been trying to figure this question out for the longest time. I also got down to answer choices A and C. I really can't figure out which conditions either of them violates.
 
timmydoeslsat
Thanks Received: 887
Atticus Finch
Atticus Finch
 
Posts: 1136
Joined: June 20th, 2011
 
This post thanked 2 times.
 
trophy
Most Thanked
trophy
First Responder
 

Re: Q19

by timmydoeslsat Thu Apr 26, 2012 6:41 pm

These are the two choices that a majority of people will come down to on this one.

The rule that has not been used to eliminate any of the choices so far is the at least 1 OJ block.

As we can see from this game, we really have a 1-5, 1-5 repeat.

In other words,

1-5 will mirror 6-10. It is impossible for the 1-5 group to look different than the 6-10 group.

So when we look at how O and J can possibly form a block, A is our only way. They would form a block at the 5-6 spot.

With C, O and J will never be able to form that block.
 
esthertan0310
Thanks Received: 0
Jackie Chiles
Jackie Chiles
 
Posts: 33
Joined: March 03rd, 2015
 
 
 

Re: Q19

by esthertan0310 Tue Nov 14, 2017 4:46 am

I use the criteria to eliminate wrong answer choices:

[*] Neither of Kruger's laps is immediately before either of Lu's
    --> Find options that contain "Krugger, Lu" in the exact order, then eliminate those options
    -->This eliminates option (E)


[*] Jacobson does not swim lap 9
    --> It can be inferred that Jacobson does not swim lap 4 either.
    --> Find options that mention "Jacobson" at the 4th lap, then eliminate those options
    --> This eliminates option (B)


[*] Ortiz's first lap is after (but not necessarily immediately after) Miller's
    --> M > O
    --> Find options that mention "Ortiz" before "Miller", then eliminate them
    --> This eliminates option (D)


Now we are left with options A and C

[*] At least one of Jacobson's lap is immediately after one of Ortiz's laps
    --> O J
    --> Find which option does not contain "Ortiz, Jacobson" in the exact order, then eliminate it.
    --> This eliminates option (C)


Now we are left with option A, which is the correct answer.